FIN 334 Final

¡Supera tus tareas y exámenes ahora con Quizwiz!

The expected rate of return and standard deviations, respectively for four stocks are given below: LMN 4%, 0% OPQ 11%, 8% RST 11%, 9% UVW 12%, 10% XYZ 12%, 8% Which stock is clearly most desirable? A) LMN B) XYZ C) UVW D) OPQ E) RST

A) LMN

Ashley believes that the price of gas and oil is about to rise and energy company profits will follow. She should invest in A) an asset allocation fund. B) an emerging market fund. C) a sector fund. D) an aggressive growth fund.

C) a sector fund.

On a net basis, funds in the financial markets are generally supplied by A) both individuals and business firms. B) business firms. C) individuals. D) the government. E) the central bank.

C) individuals.

Markhem Enterprises is expected to earn $1.34 per share this year. The company has a dividend payout ratio of 40% and a P/E ratio of 18. What should one share of common stock in Markhem Enterprises be selling for in the market? A) $24.12 B) $24.66 C) $9.65 D) $33.77 E) $14.47

A) $24.12

Madison purchased a new car for $23,000. She was allowed $3,000 for her trade in and financed the remainder over 48 months at a rate of 4% per year. How much is her monthly payment? A) $451.58 B) $943.61 C) $519.32 D) $459.15 E) $384.91

A) $451.58

Between 1900 and 2014, a typical annual return in the U.S. stock market ranged from A) -10.2% to 29.4% B) 4.5% to 14.7% C) -16.2% to 35.4% D) -14.7% to 24.9% E) 0% to 20%

A) -10.2% to 29.4%

A portfolio has a total return of 10.5%, a beta of 0.72 and a standard deviation of 6.3%. The risk free rate is 3.8%, the market return is 12.4%. Jensen's measure of this portfolio's performance is A) 0.5%. B) 4.3%. C) 7.9%. D) 9.3%.

A) 0.5%.

What is the average annual compound rate of return for a fund with the following values? A) 10.5% B) 10.8% C) 25.6% D) 32.3%

A) 10.5%

Lauren purchased a stock for $28 a share and sold it six months later for $31. While she owned the stock, Lauren received two quarterly dividends of $0.35 per share. Lauren's holding period return on this stock is A) 13.2%. B) 11.9%. C) 12.9% D) 10.7%. E) 26.4%

A) 13.2%.

A $1,000 par value, 5% annual coupon bond matures in 4 years. The bond is currently priced at $965.35 and has a YTM of 6.0%. What is the Macaulay duration? A) 3.72 years B) 3.86 years C) 2.81 years D) .43 years E) 3.95 years

A) 3.72 years

Last year, Sue purchased a closed-end mutual fund that was trading at $42 and had an NAV of $38. Sue sold the fund today when the NAV is $44 and the market price is $43. The fund paid $1 in dividends over the past year. What is the Sue's holding period return? A) 4.8% B) 7.1% C) 11.6% D) 18.4%

A) 4.8%

An investor in the 25% marginal tax bracket purchased a bond for $983, received $85 in interest, and then sold the bond for $955 after holding it for six months. The tax rate for capital gains with holding periods in excess of one year is 15%. What are the pre-tax and post-tax holding period returns? A) 5.8%; 4.3% B) 6.0%; 4.5% C) 5.8%; 4.5% D) 6.0%; 4.3%

A) 5.8%; 4.3%

Ryan purchased a bond for $980 at the beginning of 2007. He received annual interest payments of $$55 at the end of each year through 2012 when the bond was redeemed at its face value of $1,000. Compute the yield (internal rate of return) Ryan earned on his bond purchase. A) 5.91% B) 5.61% C) 5.50% D) 5.97% E) .34%

A) 5.91%

Which one of the following statements is correct concerning international funds? A) A devaluation of the dollar causes returns on foreign investments to improve from a U.S. perspective. B) International funds are considered low-risk investments. C) Balance-of-trade positions do NOT affect the rate of return from a U.S. perspective. D) Technically, global funds can only invest in foreign securities.

A) A devaluation of the dollar causes returns on foreign investments to improve from a U.S. perspective.

Which of the following is a default option for most mutual funds? A) Automatic reinvestment of dividends and interest income. B) Automatic investment of a fixed sum each month. C) Automatic conversion from aggressive to conservative funds as clients approach retirement. D) Automatic withdrawal of a fixed amount each month.

A) Automatic reinvestment of dividends and interest income.

Lipper indexes are to assess the performance of I. equity funds. II. bond funds. III. money market funds. IV. Real Estate Investment Trusts (REITs). A) I and II only B) I and III only C) I, II and III only D) I, II, III and IV

A) I and II only

Returns on exchange traded funds may come from I. capital gains. II. dividends. III. increases in the fund's premium. IV. decreases in the fund's discount. A) I and II only B) III and IV only C) I, II and III only D) I, II, III and IV

A) I and II only

The holding period return (HPR) of one's portfolio should be compared to investment goals I. to assess whether the proper rate of return is being earned for the risk involved. II. to be sure one's portfolio is outperforming the S&P 500 Index. III. to isolate any problem investments. IV. to determine when to change benchmarks from the S&P 500 to the NASDAQ Composite Index. A) I and III only B) II and IV only C) I, II and III only D) II, III and IV only

A) I and III only

Which of the following are advantages of bond funds over individual bonds? I. greater liquidity II. maturities matched to the investor's time horizon III. automatic reinvestment of interest payments IV. diversification A) I and III only B) I, III and IV only C) I, II and IV only D) II, III and IV only

A) I and III only

Which of the following are generally considered to be good investment guidelines? I. Sell any security that has become riskier than anticipated. II. Hold all securities until they produce the highest profit attainable. III. Sell securities only if the profit can be offset with a tax loss. IV. Sell any security that no longer meets the needs of the investor. A) I and IV only B) I and III only C) I, II and IV only D) I, II, III and IV

A) I and IV only

Which of the following are key inputs to determining the intrinsic value of an asset? I. the required rate of return II. future cash flows III. current stock price IV. timing of future cash flows A) I, II and IV only B) I and II only C) II, III and IV only D) I and III only E) II and IV only

A) I, II and IV only

Which of the following are reasons to consider selling an investment that is currently in a portfolio? I. The investment has met the original objective. II. Better investment opportunities currently exist. III. The outlook for the investment has improved. IV. The investment has not met expectations and no change is expected. A) I, II and IV only B) I, III and IV only C) I, II and III only D) I, II, III and IV

A) I, II and IV only

The shares of large technology companies such as IBM and Microsoft trade I. on the NYSE. II. on the NASDAQ. III. in the third market. IV. in the fourth market. A) I, II, III and IV B) II only C) I and II only D) I, II and III only E) I only

A) I, II, III and IV

Which of the following would be found on a company's balance sheet? I. Accounts receivable II. Interest expense III. Property plant and equipment IV. Total stockholders' equity A) I, III and IV only B) I, II and IV only C) I an IV only D) III and IV only E) I, II and III only

A) I, III and IV only

A type of fund that invests in real estate and/or mortgages is known as a A) REIT. B) ETF. C) sector fund. D) hedge fund.

A) REIT.

Which one of the following statements concerning the random walk hypothesis is correct? A) Random price movements support the weak form efficient market hypothesis. B) Random price movements indicate that investors can earn abnormal profits on a routine basis. C) Stock prices in general follow repetitive patterns but the actions of individual investors are random in nature. D) Stock price movements are predictable but only over short periods of time. E) none of the above

A) Random price movements support the weak form efficient market hypothesis.

The closest approximation to the real, risk-free rate of interest is A) The short-term Treasury bill rate minus the inflation rate. B) The short-term Treasury bill rate plus the inflation rate. C) The 10 year Treasury bond rate minus the 1 year Treasury bill rate. D) The 10 year Treasury bond rate minus the inflation rate. E) The differe nce between the 10 year Treasury bond rate and the S&P 500 dividend yield.

A) The short-term Treasury bill rate minus the inflation rate.

Which of the following is ideally suited to automatic investing through a payroll deduction plan? A) a dollar cost averaging plan B) a constant dollar plan C) a constant ratio plan D) a variable ratio plan

A) a dollar cost averaging plan

Investors who who accept the random walk theory should use A) a dollar cost averaging plan. B) a constant dollar plan. C) a constant ratio plan. D) a variable ratio plan.

A) a dollar cost averaging plan.

Jeremy purchased 100 shares of FB for $19 per share in September 2012 and sold them 3 years later at $91 per share. At what annual rate did the value of his investment grow? A) about 69% B) about 379% C) about 95% D) about 126% E) about 48%

A) about 69%

Mutual funds often report returns as the growth of $10,000 over a period of time. These returns assume that A) all dividends and capital gains are reinvested. B) all dividends and capital gains are withdrawn. C) all dividends and capital gains are reinvested after deductions for income taxes. D) the investor contributes money to the fund on a regular basis through an automatic investment plan.

A) all dividends and capital gains are reinvested.

Many funds advertise their returns as the amount to which $10,000 invested in the fund would have grown to over 5 or 10 years. These results assume that A) all dividends, interest and capital gains on a pre-tax basis are reinvested in the fund. B) all dividends and capital gains less a 15% deduction for taxes are reinvested in the fund. C) all dividends and interest are reinvested in the fund. D) no dividends or capital gains are reinvested in the fund.

A) all dividends, interest and capital gains on a pre-tax basis are reinvested in the fund.

Which of the following is most likely to increase in value as the result of a weakening dollar? A) an ADR for a foreign telecommunications company B) stock in a firm that depends heavily on imported raw materials C) stock in a foreign company that depends heavily on exports to the U.S. D) stock in a firm with many accounts payable in foreign currencies

A) an ADR for a foreign telecommunications company

The tendency of naive investors to buy high (after prices have risen for several periods) and sell low (after prices have dropped for several periods) can be explained by the behavioral tendency known as A) anchoring. B) loss aversion. C) familiarity bias. D) self-attribution bias. E) overconfidence.

A) anchoring.

One drawback of investing in mutual funds is the A) annual management fee. B) lack of liquidity of fund shares. C) amount required for the initial investment. D) lack of information on the performance of the fund.

A) annual management fee.

Both the holding period to qualify and the tax rate on long-term capital gains A) are subject to political pressure and change frequently. B) are adjusted for inflation every year. C) are phased out on incomes over $388,351. D) are very stable and have not changed in many years.

A) are subject to political pressure and change frequently.

The price an individual investor will pay to purchase a stock in the OTC market is the A) ask price. B) bid price. C) spread. D) broker price. E) dirty price.

A) ask price.

Fred and Martha are in their seventies and retired. Which one of the following sets of portfolio statistics might best suit their situation if their primary investment goal is current income with limited risk? A) beta of 0.83 and a dividend yield of 6.3% B) beta of 0.86, and a dividend yield of 4.6% C) beta of 1.6 and a dividend yield of 6.4% D) beta of 1.1 and a dividend yield of 5.4%

A) beta of 0.83 and a dividend yield of 6.3%

Suppose the shares of the Chickadee Corporation traded seven times in the following sequence one day last week: 46, 45.88, 45.75, 45.50, 45.63, 46, 46.13. In this case, a limit order to sell at 46 would have been executed A) between 46 and 46.13, whereas a market order to sell could have been executed anywhere between 45.50 and 46.13. B) anywhere between 45.50 and 46.13, whereas a market order to sell would have been executed only at 46. C) only at 46, whereas a market order to sell would have been executed at 46.13. D) only at 46.13, and a market order to sell would have been executed between 46 and 46.13.

A) between 46 and 46.13, whereas a market order to sell could have been executed anywhere between 45.50 and 46.13.

The Dow Jones Industrial Average (DJIA) consists of 30 stocks whose price behavior A) broadly reflects the overall price behavior of the stock market. B) is very predictable. C) typically has little correlation with the rest of the stock market. D) reflects the changes in value of manufacturing stocks only. E) leads the movements in the general economy by one to two weeks.

A) broadly reflects the overall price behavior of the stock market.

Which of the following activities would be most useful in an efficient market. A) buying and holding a diversified portfolio B) analyzing financial ratios based on accounting data C) buying only securities that have performed well in the recent past D) investing in hedge funds E) searching for patterns in charts based on stock price movements

A) buying and holding a diversified portfolio

If a constant-dollar plan portfolio is profitable over the long run, the ________ in value over time. A) conservative portion will increase B) conservative portion will remain constant C) aggressive portion will decrease D) entire portfolio will remain constant

A) conservative portion will increase

The maximum rate of return that can be earned for a given rate of interest occurs when interest is compounded A) continuously. B) annually. C) monthly. D) daily.

A) continuously.

Combining uncorrelated assets will A) decrease the overall risk level of a portfolio. B) cause the other assets in the portfolio to become positively related. C) not change the overall risk level of a portfolio. D) increase the overall risk level of a portfolio.

A) decrease the overall risk level of a portfolio.

One advantage gained by investing in a bond fund rather than in individual bonds is the A) diversification among issuers. B) most bond funds outperform their benchmarks. C) immunity from interest rate changes. D) guarantee that the bonds will be held to maturity to avoid market fluctuations.

A) diversification among issuers.

The S & P 500 Index is an appropriate benchmark for A) diversified portfolios of large company stocks. B) portfolios diversified among several asset classes such as stocks, bonds, and real estate. C) diversified portfolios with a mix of large, small, and mid-cap stocks. D) diversified portfolios of mid-cap and small company stocks.

A) diversified portfolios of large company stocks.

Income distributed by a mutual fund from which one of the following sources receives a preferential tax rate of 15%? A) dividends on common stock B) interest on bonds C) dividends from most preferred stocks D) dividends from REITs

A) dividends on common stock

For a stock investment, the dividend yield is calculated by A) dividing a stock's annual cash dividend by its price. B) dividing a stock's price by its annual cash dividend. C) multiplying a stock's semi-annual dividend by two. D) dividing the annual change in the stock's price plus its annual dividend amount by the beginning of the year price.

A) dividing a stock's annual cash dividend by its price.

The value of a Standard & Poor's Index is computed by A) dividing the sum of the current market value of all the stocks in the index by a divisor that adjusts for stock splits and scales the Index figure to a manageable size. B) dividing the sum of the closing share prices by a divisor adjusted for changes in the companies composing the Index. C) dividing the sum of the closing share prices by a fixed divisor and then multiplying the quotient by 100. D) dividing the sum of the closing share prices by a divisor that adjusts for stock splits and scales the Index figure to a manageable size. E) dividing the sum of the current market value of all the stocks in the index by a divisor adjusted for changes in the companies composing the Index.

A) dividing the sum of the current market value of all the stocks in the index by a divisor that adjusts for stock splits and scales the Index figure to a manageable size.

The Sharpe's measure for Jane Smith's investment portfolio is 0.40, while the Sharpe's measure for the market is 0.30. This information suggests that Smith's portfolio A) exhibits superior performance because its risk premium per unit of risk is above that of the market. B) exhibits poor performance because its risk premium per unit of risk is below that of the market. C) is inadequately diversified, and more securities should be added to the portfolio in order to bring it in line with the market. D) is overly diversified, and some securities should be sold to bring the portfolio in line with the market.

A) exhibits superior performance because its risk premium per unit of risk is above that of the market.

When using a constant dollar plan, A) gains from the speculative portion of the portfolio are transferred to the safe portion of the portfolio. B) the amount of money in the speculative portion of the portfolio should decline over time. C) the percentage of funds in the speculative portion of the portfolio should increase over time. D) the ratio of safe funds to speculative funds remains constant over time.

A) gains from the speculative portion of the portfolio are transferred to the safe portion of the portfolio.

Performance fees based on profits earned by the fund are typical of A) hedge funds. B) exchange traded funds. C) closed-end investment companies. D) open end mutual funds.

A) hedge funds.

People tend to A) ignore information that contradicts their current beliefs. B) earn positive abnormal profits when stock prices are rising. C) overestimate the effects of random chance. D) look at the entire situation when analyzing an individual security. E) be under-confident in their judgment of investments.

A) ignore information that contradicts their current beliefs.

A fund that is designed to match the performance of a measure such as the S & P 500 or the Russell 2000 is called a(n) A) index fund. B) targeted fund. C) sector fund. D) block fund.

A) index fund.

An open-end investment company A) is involved in all trades of its shares. B) sells shares at a discounted NAV price. C) trades like a stock on the exchanges. D) has a set number of shares.

A) is involved in all trades of its shares.

Closed-end funds are A) less liquid than open-end funds. B) best purchased when they are selling at a premium. C) purchased directly from the funds' manager. D) traded at NAV.

A) less liquid than open-end funds.

McDonald's stock is now selling for $92 per share. Kim wants to buy 100 shares but only if she can do so at $90 or less. She should place a(n) A) limit order. B) market order. C) stop order. D) odd-lot order. E) fill or kill order.

A) limit order.

Which one of the following combination of features causes bond prices to be the most volatile? A) low coupon, long maturity B) low coupon, short maturity C) high coupon, short maturity D) high coupon, long maturity

A) low coupon, long maturity

A mutual fund is generally more tax efficient when it has a ________ turnover rate and a ________ dividend yield. A) low; low B) low; high C) high; low D) high; high

A) low; low

The two primary media for warehousing liquidity are A) money market mutual funds and money market deposit accounts. B) certificates of deposit and short-term bond funds. C) certificates of deposit and long-term bond funds. D) short-term bond funds and asset allocation funds.

A) money market mutual funds and money market deposit accounts.

Treynor's measure of portfolio performance focuses on A) nondiversifiable risk. B) diversifiable risk. C) total risk. D) the standard deviation of the portfolio.

A) nondiversifiable risk.

The process of selling certain issues in a portfolio and purchasing new ones to replace them is known as A) portfolio revision. B) market timing. C) red herring baiting. D) dollar cost averaging.

A) portfolio revision.

According to the semi-strong form of the efficient market hypothesis, which of the following might lead to extraordinary profits? A) possessing private information not available to other investors B) carefully timing trades to buy when the price is low and sell when the price is high C) studying charts of a stock's past price behavior D) thoroughly analyzing the state of the economy, the industry and the company's fundamentals E) none of the above

A) possessing private information not available to other investors

A problem investment A) requires immediate attention. B) is one you would buy if you did not already own it. C) is defined as any investment with unrealized losses. D) should be left alone and given time to correct itself.

A) requires immediate attention.

The constant-ratio plan A) requires the establishment of trigger points for portfolio rebalancing. B) utilizes a predetermined ratio between desired current yield and expected capital gains. C) strictly adheres to a buy-and-hold strategy. D) is an attempt to time the cyclical movements of the market.

A) requires the establishment of trigger points for portfolio rebalancing.

Stock investors are most interested in which one of the following ratios? A) return on equity B) net profit margin C) return on assets D) equity multiplier E) current ratio

A) return on equity

A single bond issue with multiple maturity dates is called a A) serial bond. B) convertible bond. C) term bond. D) callable bond. E) premium bond.

A) serial bond.

Which stage of an industry's growth cycle is interesting only for potentially high dividend payouts? A) stability or decline B) rapid expansion C) initial development D) mature growth

A) stability or decline

Analysts commonly use the ________ to measure market return. A) the Standard & Poor's 500 Index B) Russel 1000 Index C) some large, mainstream company such as General Electric D) the Dow Jones Industrial Average E) the rate of return on 10 year Treasury bonds

A) the Standard & Poor's 500 Index

Asset allocation should focus on A) the investor's financial and family situation. B) selection of individual securities within an asset class. C) maximization of current income. D) maximization of short-term profits.

A) the investor's financial and family situation.

Bonds are least likely to be called if A) they are selling at a substantial discount. B) if they do not mature for at least 5 years. C) the price is close to par value. D) they are selling at a substantial premium.

A) they are selling at a substantial discount.

Which one of the following is a leverage measure? A) times interest earned B) return on equity C) net profit margin D) net working capital E) current ratio

A) times interest earned

Portfolios falling to the left of the efficient frontier A) would be desirable if only they were possible. B) have very high betas. C) fall within the set of feasible portfolios. D) do not use all of the assets in the portfolio. E) have too much risk for the expected return

A) would be desirable if only they were possible.

In the U. S., the most prestigious designation for financial planners is A) CFA. B) ING. C) CFP. D) CPA. E) SIPC.

C) CFP.

An asset allocation plan should consider which of the following investor characteristics? I. income and employment security II. marital status III. age and proximity to retirement IV. social relationships and peer groups A) II only B) I, II and III only C) I, III and IV only D) I, II, III and IV

B) I, II and III only

Which of the following statements about Jensen's measure are correct? I. Through its use of the capital asset pricing model, Jensen's measure automatically adjusts for market return. II. In general, the higher the Jensen's measure, the better a portfolio has performed. III. Jensen's measure is referred to as alpha. IV. A positive Jensen's measure indicates an investment has underperformed the market on a risk-adjusted basis. A) I and IV only B) I, II and III only C) II and III only D) I, III and IV only

B) I, II and III only

JJ Industries has a P/E ratio of 18 and an EPS of $0.93. This means that JJ's stock is currently selling for A) $19.35 per share. B) $16.74 per share. C) $18.00 per share. D) $18.00 per share. E) $17.07 per share.

B) $16.74 per share.

Information for ABC Fund is The market-based holding period return for ABC Mutual Fund is A) -19.1%. B) -14.6%. C) 0.3%. D) 9.4%.

B) -14.6%.

Emily bought 200 shares of ABC Co. stock for $29.00 per share on 60% margin. Assume she holds the stock for one year and that her interest costs will be $80 over the holding period. Ignoring commissions, what is her percentage return (loss) on invested capital if the stock price went down 10%? A) 10% B) -19% C) -32% D) -10% E) -16%

B) -19%

The required return on Beta stock is 14%. The risk-free rate of return is 4% and the real rate of return is 2%. How much are investors requiring as compensation for risk? A) 2% B) 10% C) 14% D) 8% E) 12%

B) 10%

Over the period 1956-2011, stocks have provided investors with annual returns between A) 5% to 7%. B) 10% to 12%. C) 3% to 5%. D) 12% to 14%. E) 7% to 9%.

B) 10% to 12%.

Ten years ago, Taylor purchased 444.44 shares in a mutual fund for $22.50 per share. He has never made an additional investment in this fund, but because of reinvested dividends and capital gains, he now owns 1,200 shares with a net asset value of $25.88 per share. Ignoring taxes, his compound average annual rate of return (IRR) is A) 10.0%. B) 12%. C) 21%. D) 31%.

B) 12%.

Nadine Enterprises has total assets of $240,000, a debt-equity ratio of 0.60, and a return on assets of 9%. What is the return on equity? A) 5.4% B) 14.4% C) 5.6% D) 22.5% E) 15.0%

B) 14.4%

Gerry bought 100 shares of stock for $30.00 per share on 70% margin. Assume Gerry holds the stock for one year and that his interest costs will be $45 over the holding period. Gerry also received dividends amounting to $0.30 per share. Ignoring commissions, what is his percentage return on invested capital if he sells the stock for $34 a share? A) 9.16% B) 18.33% C) 20.48% D) 12.83% E) 106.17%

B) 18.33%

Six months ago, Suzanne purchased a stock for $28 a share. Today she sold the stock at a price of $32 a share. During the time she owned the stock, she received a total of $1.30 in dividends per share. What is her holding period return? A) 16.6% B) 18.9% C) 33.2% D) 37.8%

B) 18.9%

The Sorka Corp. has paid annual dividends of $0.60, $0.63, $0.65, $0.68 and $0.72, respectively, over the past five years. What is the dividend growth rate? A) 5.2% B) 4.7% C) 5.4% D) 5.0% E) 5.9%

B) 4.7%

Marti is 31 years old and is saving for retirement. Which one of the following portfolio allocations might best suit her situation if she is willing to accept a fair amount of risk in exchange for long-term capital appreciation? A) 60% bonds, 15% money funds and 25% real estate B) 5% money funds, 10% bonds and 85% growth stocks C) 25% bank CDs, 40% corporate bonds, 15% money market, 20% value stocks D) 50% mortgage bonds, 5% money market, 45% municipal bonds

B) 5% money funds, 10% bonds and 85% growth stocks

Phil has a portfolio with a 13.2% total return. The beta of the portfolio is 1.48 and the standard deviation is 13%. Currently, the risk-free rate of return is 4% and the overall market has a total return of 11%. What is the value of Treynor's measure for Phil's portfolio? A) 2.1% B) 6.2% C) 7.1% D) 8.9%

B) 6.2%

A stock has a total return of 16.4%, a standard deviation of 14.5% and a beta of 1.63. The market rate of return is 12.4%, while the market's Treynor measure is 6.3. What is the value of the Treynor measure of this portfolio? A) 2.5% B) 6.3% C) 18.4% D) 27.6%

B) 6.3%

Small company stocks are yielding 10.7% while the U.S. Treasury bill has a 1.3% yield and a bank savings account is yielding 0.8%. What is the risk premium on small company stocks? A) 11.5% B) 9.4% C) 10.7% D) 12.0% E) 9.9%

B) 9.4%

On a table are 100 sacks, each of which contain 1,000 poker chips. Forty-five of these sacks contain 70% black chips and 30% red chips. The other 55 bags hold 70% red chips and 30% black chips. Suppose you choose one bag at random and then take out 12 chips. Of the 12 chips that you pull out, 8 are black and 4 are red. What is the probability that the bag you picked contains mostly black chips? A) 30% B) 95% C) 70% D) 75% E) 85%

B) 95%

Allison's portfolio has an expected return of 14% and a beta of 1.37. Brianna's portfolio has an expected rate of return of 11% and a beta of 1. The risk-free rate is 3% and the expected rate of return on the market is 12%. According to the Jensen's measure, A) Allison has the better portfolio. B) Brianna has the better portfolio. C) the portfolios are equally desirable. D) the answer depends on Allison and Brianna's risk tolerance.

B) Brianna has the better portfolio.

Allison's portfolio has an expected return of 14% and a standard deviation of of 20%. Brianna's portfolio has an expected rate of return of 11% and a standard deviation of 12%. The risk-free rate is 3%. According to the Sharpe measure, A) Allison has the better portfolio. B) Brianna has the better portfolio. C) The portfolio's are equally desirable. D) The answer depends on Allison and Brianna's risk tolerance.

B) Brianna has the better portfolio.

Which one of the following statements concerning formula plans is correct? A) The use of subjective judgment is important to the routine administration of most formula plans. B) Formula plans are based on the adherence to a mechanical set of rules with regard to when to buy and/or sell. C) The objective of most formula plans is to maximize profits. D) Securities with very stable prices are best suited to nearly all formula plans.

B) Formula plans are based on the adherence to a mechanical set of rules with regard to when to buy and/or sell.

Which of the following statements best describes the legal organization of mutual funds? A) Funds are organized as a single entity that handles all functions such as custody and investment decisions. B) Funds split their basic functions such as record keeping and investment decisions among two or more companies. C) Funds are owned by the company that manages them. D) A distributor keeps track of investment and redemption requests from shareholders and maintains other shareholder records.

B) Funds split their basic functions such as record keeping and investment decisions among two or more companies.

Which of the following statements is(are) correct concerning exchange-traded funds (ETFs)? I. You can buy and sell ETFs any time during trading hours. II. ETFs are actively managed. III. ETFs have high portfolio turnover rates. IV. ETFs rarely distribute any capital gains. A) I, II and IV only B) I and IV only C) II and III only D) I, III and IV only

B) I and IV only

Which of the following are reasons why a person may want to warehouse liquidity? I. protect against total loss II. ability to exploit future opportunities III. capitalize on the high rates of return available on cash IV. protect against the need to disturb the existing portfolio A) I and II only B) I, II and IV only C) II, III and IV only D) I, II, III and IV

B) I, II and IV only

Which of the following will tend to improve a bond's rating? I. an improvement in the firm's cash flow II. an increase in corporate debt III. an increase in net profits IV. an increase in net working capital A) II, III and IV only B) I, III and IV only C) I, II and III only D) I, II, III and IV E) III and IV only

B) I, III and IV only

Value funds seek stocks I. with low dividend yields. II. with potential for growth. III. with low P/E ratios. IV. of newly discovered firms. A) I and III only B) II and III only C) II, III and IV only D) I, II, III and IV

B) II and III only

Which of the following accurately describe reasons for investing in mutual funds? I. to effectively control the timing of capital gains for tax purposes II. to achieve portfolio diversification at a reasonable cost III. to invest in unfamiliar sectors or geographic regions IV. to outperform the market A) I and IV only B) II and III only C) I, II and III only D) I, II, III and IV

B) II and III only

James is willing to settle for a 10% rate of return on EG stock at a time when investors, on average, are requiring an 11% rate of return on the same stock. Which of the following will happen? A) James will be have to pay more for the stock than he was willing to pay. B) James will be happy to buy the stock for less than he was willing to pay. C) James will change his intrinsic value for the stock, but the direction of the change cannot be determined. D) Investors with different required rates of return will pay different prices for the stock. E) James will not be able to buy the stock unless the price changes.

B) James will be happy to buy the stock for less than he was willing to pay.

Marco's just reported an EPS of $1.68 on revenues of $440 million. The company has 12 million shares outstanding. Total assets are $280 million, current liabilities equal $48 million, and long-term debt is $112 million. Net fixed assets are worth $230 million. Given this information, which one of the following statements is correct? A) Marco's return on equity is 0.12. B) Marco's net working capital is $2 million. C) Marco's current ratio is 1.75. D) Marco's debt-equity ratio is 0.75. E) Marco's total asset turnover is 3.67.

B) Marco's net working capital is $2 million.

Which one of the following statements is correct concerning players in the mutual fund industry? A) Security analysts and traders work for the management company. B) Normally a bank serves as the custodian. C) The management company maintains the shareholder records. D) The mutual fund shareholders are the owners of the management company.

B) Normally a bank serves as the custodian.

Which of the following designations does NOT have formal education and testing requirements? A) Chartered Financial Analyst B) Registered Investment Adviser C) Certified Public Accountant D) Certified Financial Planner

B) Registered Investment Adviser

Which one of the following indexes reflects a large sample of small, medium sized and large companies? A) NYSE composite B) Russell 3000 C) Value Line composite D) DJIA E) S&P MidCap 400

B) Russell 3000

If an investor's portfolio is comprised of a broad range of common stocks, the best measure to use as a basis of comparison of performance is the A) Dow Jones Industrial Average (DJIA). B) S&P 500 index. C) Dow Jones Corporate Bond Index. D) American Stock Exchange utilities index.

B) S&P 500 index.

Aria has heard a rumor that a major food company will be forced to recall millions of jars of peanut butter due to contamination. If the rumors are true, the company's stock price will decline sharply. Which one of the following strategies would allow Jennifer to earn a profit if the rumor proves to be true? A) Buy a call option on the stock that expires next month. B) Sell the stock short today. C) Take a long position in the stock one month from today. D) Take a long position in the stock today. E) Buy the stock on margin today.

B) Sell the stock short today.

Which one of the following statements concerning ETFs is correct? A) ETFs are based solely on U.S. indexes. B) The ETF based on the Standard & Poor's 500 Index is priced at 1/10 the value of that index. C) Spiders are based on the DJIA. D) The ETF based on the Dow is priced at 1/10 of the value of the DJIA.

B) The ETF based on the Standard & Poor's 500 Index is priced at 1/10 the value of that index.

Which of the following is published yearly by publicly-held corporations and provided free to their stockholders and other investors? A) brokerage reports B) annual reports C) tombstones D) back-office reports E) red herrings

B) annual reports

Which one of the following statements concerning the primary market is correct? A) A rights offering is a direct sale of stock to an institution that participates in the primary market. B) The first public sale of a company's stock is called an IPO. C) The first public sale of a company's stock in the primary market is called a seasoned new issue. D) A transaction in the primary market is between two private stockholders. E) The primary market operates from 9:30am to 4:00pm every day.

B) The first public sale of a company's stock is called an IPO.

Which one of the following statements is correct if a portfolio has a Jensen measure of return of zero? A) The portfolio has a total return of zero percent. B) The portfolio earned exactly its expected return on a risk-adjusted basis. C) The portfolio outperformed the market on a risk-adjusted basis. D) The market provides a better return on a risk-adjusted basis.

B) The portfolio earned exactly its expected return on a risk-adjusted basis.

Which one of the following is likely to have a negative effect on stock prices? A) rising home prices B) a decrease in the money supply (M2) C) falling interest rates D) low inflation E) a decrease in the unemployment rate

B) a decrease in the money supply (M2)

One important tax rule concerning capital losses is that A) capital losses are always fully deductible. B) a maximum of $3,000 of losses in excess of capital gains can be written off against ordinary income in any one year. C) a maximum of $10,000 of losses in excess of capital gains can be written off against other income in any one year. D) capital losses are never deductible.

B) a maximum of $3,000 of losses in excess of capital gains can be written off against ordinary income in any one year.

Mike bought 200 shares of EG stock two years ago at $16 per share. The stock has traded in a range of $21 to $44 a share over the past year. EG is now selling for $43.60 a share. EG announces its earnings today and Mike feels the stock could go to $60 on good news or fall to $30 on bad. To protect his profits, the most appropriate order for him to place is A) market order to sell immediately. B) a stop loss order at $42. C) a stop loss order at $16. D) a stop-limit order to sell at $45. E) a limit sell order at $60.00.

B) a stop loss order at $42.

Which one of the following will tend to cause domestic interest rates to rise? A) a decline in the real rate of return B) an increase in interest rates overseas C) a decrease in the rate of inflation D) an increase in the money supply E) a decrease in the federal budget deficit

B) an increase in interest rates overseas

Municipal bonds can be either general obligation bonds or revenue bonds. Of these two types of municipal bonds, only general obligation bonds A) have the principal and interest guaranteed by a third party. B) are backed by the full faith and credit of the issuer. C) offer tax-exempt interest income. D) are specifically serviced by the income generated from particular projects. E) repay the principal only if a sufficient level of revenue is generated.

B) are backed by the full faith and credit of the issuer

Back-end loads A) are charged when an investor buys their mutual fund shares. B) are charged if an investor sells his or her shares within the first few years. C) were designed to help no-load funds cover their marketing expenses. D) encourage short-term trading.

B) are charged if an investor sells his or her shares within the first few years.

Which one of the following provides the greatest reduction in total risk? A) diversification B) asset allocation C) security selection D) beta reduction

B) asset allocation

The ability to automatically buy additional fund shares using the dividend income generated by the fund is called a(n) A) automatic investment plan. B) automatic reinvestment plan. C) systematic withdrawal plan. D) conversion plan.

B) automatic reinvestment plan.

A high TRIN value is considered A) good for the market when the number of advancing stocks is declining. B) bad for the market when the trading volume in the declining stocks is rising. C) bad for the market when the number of declining stocks is stable. D) good for the market when the volume of advancing stocks is declining.

B) bad for the market when the trading volume in the declining stocks is rising.

Which type of mutual fund consists of both stocks and bonds with a combined objective of current income and long-term capital gains? A) equity-income B) balanced C) value D) bond

B) balanced

The conversion privilege provided by mutual fund families allows investors to A) move from one fund to another without incurring any capital gains tax liability. B) be more aggressive since they can re-allocate their funds when market conditions change. C) re-allocate their funds at any time as long as they pay an additional sales load on the transferred funds. D) move from one fund family to another once every six months.

B) be more aggressive since they can re-allocate their funds when market conditions change.

Excessively trading a customer's account to increase a stockbroker's commission income is A) an acceptable method of timing the market to increase rates of return. B) called churning which is an illegal practice. C) permitted provided that the customer does not object. D) probably unethical but yet is acceptable by the securities industry

B) called churning which is an illegal practice.

Investment companies can inflate the percentage of funds that outperformed their benchmark by A) including unrealized capital gains. B) closing funds that underperformed their benchmark. C) using a different fiscal year than the benchmark. D) assuming that investors automatically add to their holdings.

B) closing funds that underperformed their benchmark.

The primary objective of an equity-income fund is A) capital gains. B) current income with capital preservation. C) potentially high capital gains with limited income. D) high risk-return trade-offs.

B) current income with capital preservation.

One characteristic of 12(b)-1 charges is that they are payable A) only in years that the mutual fund shows an increase in net asset value. B) each year regardless of the performance of the mutual fund. C) only during the first year the fund is owned. D) only when shares in the fund are sold.

B) each year regardless of the performance of the mutual fund.

Jocelyn sells short 1000 shares of JKLO stock at $31.25 per share and six months later purchases the shares at $29.00 each. Ignoring brokerage fees, Nancy will A) lose a total of $2,900. B) earn a total profit of $2,250. C) earn a total profit of $3,125. D) lose a total of $3,125. E) lose a total of $2,250.

B) earn a total profit of $2,250.

One characteristic of most index funds is that such funds typically A) produce a large dollar amount of realized capital gains every year. B) have a very low-cost structure with respect to management fees and transaction fees. C) charge high front-end loads. D) are designed to "beat the market."

B) have a very low-cost structure with respect to management fees and transaction fees.

According to chartists, a breakout below a support level A) is a buy signal but only for value investors. B) is a sell signal. C) is a signal that the market is stagnant. D) is a buy signal.

B) is a sell signal.

When a corporation declares a stock split, it usually does so because A) it wants to boost its market capitalization. B) it wants to make its stock more affordable to average investors. C) investors sometimes require nontaxable returns. D) the firm's retained earnings are excessive. E) there are too many shares of stock outstanding.

B) it wants to make its stock more affordable to average investors.

Investors in hedge funds have the legal status of A) shareholders. B) limited partners. C) general partners. D) trustees.

B) limited partners.

Which one of the following is a characteristic of blue chip stocks? A) relatively high risk exposure B) long and stable dividend and earnings records C) headquarters location near the ocean D) annual dividends of more than $5 per share E) guaranteed minimum annual dividend of $2 a share

B) long and stable dividend and earnings records

Over the long term, which one of the following has historically had the lowest risk and lowest average annual rate of return? A) corporate bonds B) long-term government bonds C) real-estate D) mortgage-backed securities E) common stock

B) long-term government bonds

Mutual fund investors are primarily exposed to ________ and ________ risks. A) market; financial B) market; inflation C) business; financial D) business; inflation

B) market; inflation

Income from ________ is exempt from federal income taxes? A) money market funds B) municipal bond funds C) government bond funds D) REITs

B) municipal bond funds

Investors seeking a diversified, professionally managed portfolio of securities can purchase shares of A) insurance policies. B) mutual funds. C) common equity. D) convertible securities. E) preferred stock.

B) mutual funds.

Eurodollar bonds are A) purchased with dollars but redeemed in euros. B) purchased and redeemed in dollars but issued by entities outside the U.S. C) purchased with dollars, but redeemable in either euros or dollars. D) currently paying negative interest rates. E) purchased with euros but redeemed in dollars.

B) purchased and redeemed in dollars but issued by entities outside the U.S.

The fixed-weightings approach to asset allocation A) is based on an allocation of an equal percentage of the portfolio to each separate asset category. B) requires periodic rebalancing of the portfolio to maintain the desired weights. C) is based on periodic adjustments to category weights in response to market changes. D) uses stock-index futures and bond futures in a market timing strategy.

B) requires periodic rebalancing of the portfolio to maintain the desired weights.

Funds that invest in a portfolio of companies from the same or closely related industries are known as A) aggressive growth funds. B) sector funds. C) emerging market funds. D) asset allocation funds.

B) sector funds.

The practice of bundling mortgages or other types of loans into pools and selling pieces of the pool as bond-like instruments to investors is known as A) fractionalization. B) securitization. C) privatization. D) marginalization. E) collateralization.

B) securitization.

Investors who wish to minimize the effect of taxes on their investment returns should try to avoid A) dividend paying stocks. B) short-term capital gains. C) long-term capital gains. D) municipal bonds.

B) short-term capital gains.

Under which bond provision is the issuer required to retire portions of the bond issue prior to maturity? A) subordination clause B) sinking fund feature C) trust certificates D) refunding provision E) call feature

B) sinking fund feature

Investors interested in predictable cash flow from their investments should consider funds that offer A) conversion privileges. B) systematic withdrawal plans. C) automatic reinvestment plans. D) automatic investment plans.

B) systematic withdrawal plans.

Stocks related to computers and the Internet are classified as A) blue-chip stocks. B) tech stocks. C) income stocks. D) vlaue stocks. E) cyclical stocks.

B) tech stocks.

The over-the-counter (OTC) market is a A) traditional broker market. B) telecommunications network connecting dealers. C) geographically dispersed auction market. D) market solely for institutional traders. E) centrally located auction market.

B) telecommunications network connecting dealers.

The primary risk in using a GTC limit sell order rather than a market order is that A) the market price may exceed the limit price when the order is placed. B) the limit order may not be executed. C) the limit order may be executed at a price above the market price. D) the limit order expires at the end of the day and may not be executed.

B) the limit order may not be executed.

Sharpe's measure of portfolio performance compares the risk premium on a portfolio to A) a broad-based market index such as the S&P 500 index. B) the portfolio's standard deviation of return. C) the portfolio's beta. D) the prevailing risk-free rate of return.

B) the portfolio's standard deviation of return.

The required return on a bond is equal to A) the risk-free rate plus a risk premium plus an expected inflation premium. B) the real rate of return plus a risk premium plus an expected inflation premium. C) the real rate of return plus the coupon rate plus an inflation rate. D) the real rate of return plus an expected inflation premium. E) the real rate plus a risk premium.

B) the real rate of return plus a risk premium plus an expected inflation premium.

The theory behind the variable ratio plan is to A) passively buy and hold a wide variety of securities. B) time the cyclical movements of the stock market and thereby "buy low and sell high." C) avoid selling any security for a capital gain, and thus indefinitely avoiding the capital gains tax. D) keep the unit cost of the portfolio at a constant level.

B) time the cyclical movements of the stock market and thereby "buy low and sell high."

The general theory of dollar cost averaging is A) to time the market to take advantage of low stock prices. B) to buy more stock when prices are low and less when prices are high. C) to equal the performance of market averages at the lowest dollar cost. D) to sell as markets decline and buy as they begin to rise.

B) to buy more stock when prices are low and less when prices are high.

David has purchased an investment that he expects to produce an annual cash flow of $3,000 for five years. He requires an 8% rate of return compounded annually. What is the maximum amount that David can pay and still earn the required rate of return? A) $12,936 B) $14,764 C) $11,978 D) $19,008 E) $15,000

C) $11,978

Juan's investment portfolio was valued at $125,640 at the beginning of the year. During the year, Juan received $603 in interest income and $298 in dividend income. Juan also sold shares of stock and realized $1,459 in capital gains. Juan's portfolio is valued at $142,608 at the end of the year. All income and realized gains were reinvested. No funds were contributed or withdrawn during the year. What is the amount of income Juan must declare this year for income tax purposes? A) $0 B) $901 C) $2,360 D) $19,328

C) $2,360

Zephyr Inc. sells wind based systems for generating electricity. The company pays no dividends, but you estimate the stock will be worth $50 per share 5 years from now and you require a 15% rate of return for stock investments of this type. What price should you be willing to pay for this stock? A) $28.59 B) $43.48 C) $24.86 D) $12.50 E) $57.50

C) $24.86

For the period 2000 through 2009, the average annual price change for stocks in the S&P 500 index was A) 16%. B) 8%. C) -1%. D) -50%. E) 9.6%.

C) -1%.

Ella owns a stock with a beta of 1.34 and a standard deviation of 16.4%. The stock has a total return of 14.8%. The market risk premium is 8.5%, while the return on the market portfolio was 12.0%. What is the value of Sharpe's measure for Ella's portfolio? A) 0.21 B) 0.38 C) 0.69 D) 0.90

C) 0.69

Tim purchased a stock ten months ago for $14 a share, received a $1 dividend per share last month, and sold the stock today for $16 per share. Tim has a marginal tax rate of 30%. Both capital gains for securities held more than one year and dividend income is taxed at 15%. What is Tim's after-tax holding period return? A) 14.1% B) 15.9% C) 16.1% D) 18.2%

C) 16.1%

On January 1, Stacy's portfolio was valued at $96,534. During the year Stacy received $3,285 in interest and $4,100 in dividends. She also sold one stock at a gain of $850. The value of the portfolio on December 31 of the same year was $113,201. At the end of June, Stacy withdrew $5,000 from the portfolio. What is the holding period return for the year? A) 25.1% B) 25.8% C) 26.5% D) 27.2%

C) 26.5%

GLOO stock's P/E ratio is 45 at a time when the market's P/E ratio is 15. GLOO's relative P/E ratio is A) .33. B) 30. C) 3. D) 1.5. E) -30.

C) 3.

The required rate of return on the Cosmos Corporation's common stock is 10%, the current real rate of return in the market is 1%, and the inflation rate is 3%. In this case, the risk premium associated with Cosmos stock is A) 5%. B) 10% C) 6%. D) 8%. E) 7%.

C) 6%.

A portfolio has a total return of 14.5%, a beta of 1.54, and a standard deviation of 17.6%. If the risk free rate is 4.5% and the market return is 10.2%, then Treynor's measure of this portfolio's performance is A) 2.8%. B) 3.7%. C) 6.5%. D) 9.4%.

C) 6.5%.

Maria purchased $5,000 of no-load mutual fund shares just over a year ago. She received $136 in dividend income and $201 in long-term capital gains distributions. Today she sold her shares for $5,062. Maria is in the 25% marginal tax bracket. Capital gains with holding periods in excess of one year and dividend income are taxed at 15%. What is Maria's after-tax holding period return? A) 6.0% B) 6.6% C) 6.8% D) 8.0%

C) 6.8%

The yield-to-maturity (YTM) approach fails to consider which of the following risks? I. reinvestment risk II. price or market risk A) II only B) I only C) Both I and II D) Neither I nor II

C) Both I and II

Two mutual funds are quoted as follows. Given these quotes, which one of the following is true? A) Both funds are load funds. B) Fund A is a no-load fund. C) Fund B is a no-load fund. D) Both funds are no-load funds.

C) Fund B is a no-load fund.

Which of the following theories is consistent with yield curves sloping upward most of the time? I. market segmentation theory. II. expectations theory. III. liquidity preference theory. IV. theory of evolution. A) I, II and III only B) I, II, III and IV C) I and III only D) II, III and IV only E) III and IV only

C) I and III only

The following calendar describes some relevant dates for an upcoming dividend distribution of $1.50. In the absence of any new relevant information about the firm: I. If the stock traded for $25 at the close of trading on the 5th, it would most likely begin trading at $23.50 on the 6th. II. If the stock traded for $25 at the close of trading on the 6th, it would most likely begin trading at $23.50 on the 7th. III. All investors that purchased the stock before the close of trading on the 7th would receive the dividend. IV. All investors that purchased the stock before the close of trading on the 6th would receive the dividend. A) I and IV B) II and III only C) I only D) II only E) IV only

C) I only

A moderate asset allocation alternative might include I. bonds. II. common stocks. III. foreign securities. IV. options and commodities futures. A) I and II only B) I, III and IV only C) I, II and III only D) I, II, III and IV

C) I, II and III only

Which of the following characteristics apply to closed-end mutual funds? I. unlimited number of outstanding shares II. transactions between shareholders III. Market prices may be higher or lower than NAV. IV. Fund will repurchase shares at any time. A) I and IV only B) II and III only C) I, II and III only D) II, III and IV only

C) I, II and III only

Reasons to invest in mutual fund include I. a wide range of services such automatic reinvestment and systematic withdrawal plans. II. minimizing the time and effort spent choosing securities. III. rates of return that consistently beat the market averages. IV. participation in a variety of tax sheltered and tax deferred retirement programs. A) I and II only B) II, III and IV only C) I, II and IV only D) I, II, III and IV

C) I, II and IV only

Typical services offered by mutual funds include: I. Automatic reinvestment plans. II. Automatic investment plans. III. Automatic fund conversion as investment goals change. IV. Automatic withdrawal plans A) I and III only B) II and IV only C) I, II and IV only D) I, II, III and IV

C) I, II and IV only

Which of the following are characteristics of stop-loss orders? I. the risk of whipsawing II. the ability to limit downside losses III. the guaranteed execution within the order period IV. the conversion to a market order A) I and II only B) III and IV only C) I, II and IV only D) II, III and IV only

C) I, II and IV only

Which of the following factors are considered when analyzing an industry? I. the nature and conditions of governmental regulations II. the involvement and relations, if any, with labor unions III. the development of new technologies relevant to the industry IV. the extent of competition within the industry A) II, III and IV only B) I, II and III only C) I, II, III and IV D) III and IV only E) I, II and IV only

C) I, II, III and IV

Which of the following factors are included in the rating analysis of a corporate bond? I. the issue's indenture provisions II. the liquidity position of the issuing company III. the issuing company's relative debt burden IV. the stability of the company's earnings A) I and II only B) I, III and IV only C) I, II, III and IV D) II, III and IV only E) I, II and III only

C) I, II, III and IV

Advantages of investing in real estate potentially include I. rental income. II. liquidity. III. tax write offs. IV. capital gains. A) I and IV only B) II and IV only C) I, III and IV only D) II and III only E) I, II, III and IV

C) I, III and IV only

Which of the following are sources of income for an open-end mutual fund? I. dividend and interest income II. change in the discount or premium III. capital gains IV. change in NAV A) I and III only B) I, II and III only C) I, III and IV only D) II, III and IV only

C) I, III and IV only

Which of the following factors influence short-term interest rates on government securities? I. Federal Reserve actions II. interest rate risk III. expected future inflation IV. the real rate of return A) I, II and IV only B) III and IV only C) I, III and IV only D) I and III only E) II and IV only

C) I, III and IV only

Which of the following statements concerning the Price to Cash-Flow approach to stock valuation are true? I. The Price to Cash-Flow method works just as well for non-dividend paying stocks as it does for dividendpaying stocks. II. The Price to Cash-Flow calculates the intrinsic value of a stock as the present value of future cash flows. III. The Price to Cash-Flow ratio divides the market price of one share of stock by cash flow per share. IV. The Price to Cash-Flow method does not directly calculate the intrinsic value of a share. A) I and II only B) I, II and III only C) I, III and IV only D) III and IV only E) II and IV only

C) I, III and IV only

Which of the following statements is(are) correct concerning hedge funds? I. They are highly regulated. II. They hedge all positions to limit risks. III. Management and other fees are extremely low compared to other types of funds. IV. Access is limited to institutions and high net worth or high income individuals. A) I, II and III only B) II and IV only C) IV only D) I, II, III and, IV

C) IV only

Which one of the following statements about margin trading is correct? A) Margin traders are willing to accept lower return to reduce their risk. B) Margin traders are pessimistic about the future price of the stock. C) If Fred buys $1,000 worth of stock using 60% margin, he will need to pay $600 in cash to make the purchase. D) The Securities Exchange Commission sets the minimum margin requirement for margin trading. E) If Fred buys $1,000 worth of stock using 60% margin, he will need to pay $400 in cash to make the purchase.

C) If Fred buys $1,000 worth of stock using 60% margin, he will need to pay $600 in cash to make the purchase.

Which ratio is commonly used to distinguish between value and growth firms? A) Price-to-sales ratio B) Current ratio C) Market-to-book ratio D) Price-to-users ratio E) Price-to-cash-flow ratio

C) Market-to-book ratio

Which one of the following statements concerning mutual funds is correct? A) The selection of individual securities remains with the mutual fund investor. B) Mutual funds were first created in the 1980s. C) The mutual fund industry is the largest financial intermediary in the United States. D) Mutual funds are generally highly concentrated portfolios.

C) The mutual fund industry is the largest financial intermediary in the United States.

Allison's portfolio has an expected return of 14% and a beta of 1.37. Brianna's portfolio has an expected rate of return of 11% and a beta of 1. The risk-free rate is 3%. According to the Treynor measure, A) Allison has the better portfolio. B) Brianna has the better portfolio. C) The portfolio's are equally desirable. D) The answer depends on Allison and Brianna's risk tolerance.

C) The portfolio's are equally desirable.

A stop loss order may not be advisable A) when an investor wants to protect a gain. B) an investor wants is expecting an earnings announcement that may either exceed or fall short of expectations. C) a stock exhibits wide and rapid price fluctuations. D) when an investor wants to sell in the near future but still profit from any short-term price increases.

C) a stock exhibits wide and rapid price fluctuations.

Risk-seeking investors seeking maximum capital appreciation with little, if any current income, should invest in A) value funds. B) growth funds. C) aggressive growth funds. D) equity-income funds.

C) aggressive growth funds.

To operate as a regulated investment company and enjoy the related tax benefits, a mutual fund must annually distribute to its shareholders A) half of its realized capital gains, and interest and dividend income. B) none of its realized capital gains, but all of its interest and dividend income. C) all of its realized capital gains, and at least 90 percent of its interest and dividend income. D) all of its realized capital gains and interest and dividend income.

C) all of its realized capital gains, and at least 90 percent of its interest and dividend income.

A margin account A) can be opened by any investor who wants to purchase securities by charging them to his/her credit card. B) allows an investor to borrow one hundred percent of the cost of the securities purchased. C) allows an investor to borrow a portion of the purchase price at a reasonable rate of interest. D) allows an investor to buy and sell option contracts. E) is permitted only in wrap accounts.

C) allows an investor to borrow a portion of the purchase price at a reasonable rate of interest.

A stop loss order may not protect an investor's profits if A) the price drops even slightly below the stop price before the order can be executed. B) the price enters a prolonged period of gradual decline. C) an unexpected event cause the price to drop steeply when the markets are closed. D) the stop loss price is never reached.

C) an unexpected event cause the price to drop steeply when the markets are closed.

The confidence index indicates A) investors' trust in financial advisors. B) stock investors ' perceptions of risk in the economy. C) bond investors ' perceptions of risk in the economy. D) consumers' perceptions of risk in the economy. E) a security's strength of advances and declines over time

C) bond investors ' perceptions of risk in the economy.

Investors are generally well advised to avoid mutual funds with A) highly rated fund managers. B) low fees and high tax efficiency. C) consistently poor historical performance. D) good performance in both up and down markets.

C) consistently poor historical performance.

The maximum rate of return that can be earned for a given rate of interest occurs when interest is compounded A) annually. B) monthly. C) continuously. D) daily.

C) continuously.

Government securities money funds are structured to eliminate A) interest rate risk. B) inflation risk. C) default risk. D) market risk.

C) default risk.

The normal sequence in performing top down analysis is A) market conditions, risk, company fundamentals. B) profitability, efficiency, liquidity. C) economy, industry, company. D) competition, consumer demand, threat of substitute products.

C) economy, industry, company.

One characteristic of bond funds is the A) requirement of a minimum initial investment of $5,000 or more. B) high anticipated short-term growth potential. C) fluctuation in value in response to changing interest rates. D) extremely aggressive trading approach.

C) fluctuation in value in response to changing interest rates.

The commission charged when shares of an open-end mutual fund are purchased is called a A) management fee. B) back-end load. C) front-end load. D) 12(b)-1 fee.

C) front-end load.

Stock market averages and indexes are commonly used to measure the A) specific behavior of alternative investments. B) specific behavior of the economy. C) general behavior of stock prices. D) specific behavior of companies.

C) general behavior of stock prices.

A bond quoted at a price of 101.2 A) would cost $1,010.93. B) yields 12%. C) has a coupon rate that exceeds the market rate. D) is a deep discount bond. E) yields 10.12%.

C) has a coupon rate that exceeds the market rate.

Socially responsible funds are distinguished from other mutual funds because they A) invest only in over-the-counter stocks. B) do not charge any sales commission or management fees. C) invest only in companies that meet specified moral, ethical, or environmental standards. D) will sell their shares only to investors who sign a statement saying they do not smoke tobacco or use alcohol.

C) invest only in companies that meet specified moral, ethical, or environmental standards.

Dollar cost averaging is a procedure by which an investor A) buys more stock as its price increases. B) times investments in order to buy low and sell high. C) invests a fixed dollar amount in a security at fixed intervals. D) maintains a constant ratio of conservative and aggressive investments.

C) invests a fixed dollar amount in a security at fixed intervals.

The net asset value of a mutual fund increased from $12.03 to $13.53, but its price per share increased by only $1.26. This information indicates that the fund A) paid out $1 in capital gains. B) paid out $1 in dividends. C) is a closed-end fund. D) is an open-end fund.

C) is a closed-end fund.

A unit investment trust A) engages in short-term trading within a particular sector. B) offers a low-cost, diversified portfolio. C) is an unmanaged portfolio of securities. D) is used only for fixed-income securities.

C) is an unmanaged portfolio of securities.

Tactical asset allocation is most suitable for A) young, single individuals with good incomes. B) retirees. C) large institutional investors. D) investors who have already accumulated a fair amount of wealth.

C) large institutional investors.

Holding securities in street name A) allows the brokerage firm to sell securities without the customers approval. B) allows wealthy investors to avoid paying taxes on dividends. C) makes the trading of securities easier and more efficient for individual investors. D) enables the brokerage firm to collect the stock dividends as compensation for their services. E) means that the brokerage firm actually owns the securities.

C) makes the trading of securities easier and more efficient for individual investors.

Which stage of an industry's growth cycle is most influenced by economic events? A) rapid expansion B) incubation C) mature growth D) initial development E) stability or decline

C) mature growth

An informal, voluntary agreement to solve disputes between an investor and his/her broker by utilizing a person to facilitate negotiations between the two parties is called A) litigation. B) tribunal. C) mediation. D) binding arbitration. E) voluntary arbitration.

C) mediation.

Investors seeking tax-exempt income should invest in A) index funds. B) government bond funds. C) municipal bond funds. D) money market funds.

C) municipal bond funds.

The weak form of the efficient market theory contends that A) any publicly available information is useless in predicting future price movements. B) price movements are not random but follow a general trend over a period of time. C) past price performance is useless in predicting future price movements. D) past performance can help determine the general direction of future price movements

C) past price performance is useless in predicting future price movements.

The document that describes the issuer of a security's management and financial position is known as a A) balance sheet. B) red herring. C) prospectus. D) 8-K filing. E) 10-K report

C) prospectus.

When the Capital Asset Pricing Model is depicted graphically, the result is the A) coefficient of variation line. B) alpha-beta line. C) security market line. D) efficient frontier. E) standard deviation line.

C) security market line.

A stock's beta value is a measure of A) interest rate risk. B) total risk. C) systematic risk. D) manageable risk. E) diversifiable risk.

C) systematic risk.

If a corporation declares a 10% stock dividend, then A) the share price of the stock will most likely increase by about 10%. B) investors must pay income taxes on the distribution. C) the share price of the stock will most likely decline by about 9%. D) the share price of the stock will most likely remain unchanged. E) each shareholder will get a 10% cash rebate off his or her next round lot purchase of the stock

C) the share price of the stock will most likely decline by about 9%.

A mutual fund's net asset value is determined by A) demand for the fund based on the fund's performance. B) the fund manager's reputation. C) the value of the securities held by the fund. D) all of the above.

C) the value of the securities held by the fund.

Nearly all mutual funds operate as regulated investment companies. This means that A) they are no-load funds. B) portfolio decisions are mandated by government authorities. C) they do not pay taxes on their income. D) their investments are guaranteed by the FDIC.

C) they do not pay taxes on their income.

Investment bankers who join together to share the financial risk associated with buying an entire issue of new securities and reselling them to the public is called a(n) A) tombstone group. B) IPO team. C) underwriting syndicate. D) primary market group. E) selling group.

C) underwriting syndicate.

4) Josh earned $82,500 in taxable income, all from wages and interest, and files an individual tax return. What is the amount of Josh's taxes for the year 2012? Round to the nearest dollar. A) $13,750 B) $20,625 C) $18,425 D) $16,665 E) $25,492

D) $16,665

The current annual sales of Flower Bud, Inc. are $178,000. Sales are expected to increase by 4% next year. The company has a net profit margin of 5% which is expected to remain constant for the next couple of years. There are 10,000 shares of common stock outstanding. The market multiple is 16.4 and the relative P/E of the firm is 1.21. What is the expected market price per share of common stock for next year? A) $15.18 B) $17.66 C) $20.16 D) $18.37 E) $19.29

D) $18.37

Hallowell Inc. has free cash flow of $2.5 million and 1.25 million shares outstanding. If you believe the price to cash flow ratio for this company should be 11, what is the highest price you should pay for the stock? A) $22.72 B) $2.00 C) $27.50 D) $22.00 E) $13.75

D) $22.00

If a firm has a 2 million shares outstanding and its stock trades at $25 per share, the company also has $10,000,000 in debt. The company's market capitalization is A) $60,000,000. B) $40,000,000. C) $49,000,000. D) $50,000,000. E) the answer cannot be determined with the information given.

D) $50,000,000.

On January 1, Tim's portfolio was valued at $432,098. During the year Tim received $10,563 in interest and $15,060 in dividends. He also sold stock at a net loss of $12,870 and used the proceeds to purchase another stock. Tim did not contribute any more funds nor withdraw any funds during the year. On December 31 of the same year, Tim's portfolio was valued at $398,189. What is the holding period return for the year? A) -5.3% B) -4.9% C) -2.1% D) -1.9%

D) -1.9%

Amanda has the following portfolio of assets. What is the beta of Amanda's portfolio if the risk-free rate is 1.5% and the market risk premium in 6%? A) 0.85 B) 1.13 C) 0.733 D) 0.62 E) 2.20

D) 0.62

Spencer Inc.'s stock recently sold for $33.83 per share. Current earnings per share are $2.05 and the expected EPS 5 years from today are $ 3.64. Spencer's PEG ratio is A) 2.19. B) 1.07. C) 2.78. D) 1.36. E) 16.50.

D) 1.36.

On February 19, 2004, Angela purchased 100 shares of ABC stock at a total cost of $1,712.50. She received a total of $125.00 in dividends and sold the stock today, February 22, 2005. Her net proceeds from the sale are $1,892.40. Angela has a combined state and federal marginal tax rate of 32%. Her combined state and federal tax rate on both her capital gains in excess of one year and her dividend income is 18%. What is Angela's after-tax holding period return on her investment in ABC stock? A) 11.0% B) 12.1% C) 13.2% D) 14.6%

D) 14.6%

Ten months ago, Junior purchased a stock for $14 a share. The stock pays a quarterly dividend of $0.50 per share. Today, Junior sold the stock for $15 a share. What is his holding period return? A) 10.0% B) 10.7% C) 16.7% D) 17.9%

D) 17.9%

Compared to yields on general purpose money funds, the yields on tax-exempt money funds are A) about the same. B) 5 to 10 percent higher. C) 5 to 10 percent lower. D) 20 to 30 percent lower.

D) 20 to 30 percent lower.

Last year at this time, a mutual fund had an NAV of $13.20 per share. Over the past year the fund paid dividends of $0.70 per share and had a capital gains distribution of $1.20 per share. What is the holding period return assuming that the current NAV is $14.42? A) 13.2% B) 14.4% C) 21.6% D) 23.6%

D) 23.6%

As a general rule, which one of the following statements concerning the various values of common stock is correct? A) Intrinsic values are usually below par values. B) Market values are usually below book values. C) Par values are usually above book values. D) Book values are usually below market values. E) Market values are usually below par values

D) Book values are usually below market values.

Which of the following measures is based on the capital asset pricing model? A) Only Sharpe's measure B) Only Treynor's measure C) Only Jensen's measure D) Both Treynor's and Jensen's measures

D) Both Treynor's and Jensen's measures

EAFE stands for A) Europe, Asia, Far East. B) England, America, France, European Community. C) England, America, Far East. D) Europe, Australia, Far East.

D) Europe, Australia, Far East.

One type of mutual fund spreads investors' money across equity markets, bond markets, and money markets. Moreover, as market conditions change, the amount of money invested in each market sector will change. This type of mutual fund is known as a(n) A) socially responsible fund. B) fiscally responsible fund. C) growth-and-income fund. D) asset allocation fund.

D) asset allocation fund.

Which of the following characteristics apply to collateralized mortgage obligations? I. CMOs were created to diffuse prepayment uncertainty. II. CMOs are derivative securities created from mortgage-backed securities. III. All principal payments are paid to the shortest remaining tranche. IV. CMOs have definite maturity dates for each tranche. A) I and II only B) II, III and IV only C) II and IV only D) I, II and III only E) I and III only

D) I, II and III only

Which of the following are advantages offered by mutual funds? I. professional portfolio management II. dividend reinvestment III. consistent returns in excess of the overall market rate of return IV. modest capital outlay for investors A) I and II only B) I and IV only C) II, III and IV only D) I, II and IV only

D) I, II and IV only

Which of the following are common but dysfunctional investor behaviors? I. overinvesting in companies with familiar names II. dividing their funds equally among available choices, even if several of the choices serve the same purpose III. holding on to a stock that has dropped in value because you would be willing to buy it at its current price IV. overestimating one's ability to pick successful investments A) II and III only B) I and IV only C) I, II, III and IV D) I, II and IV only E) II and IV only

D) I, II and IV only

Which of the following are considered in the company analysis phase of a fundamental analysis of a firm? I. the composition and growth in sales II. the capital structure of the firm III. the outlook of the national economy IV. the composition and liquidity of the company's assets A) I, II, III and IV B) I and II only C) II and IV only D) I, II and IV only E) I and III only

D) I, II and IV only

Which of the following characteristics apply to exchange traded funds (ETFs)? I. unlimited number of outstanding shares II. typically track the performance of some index III. Market prices may be higher or lower than NAV. IV. May invest in the whole index or use a sample of securities to track the index as closely as possible. A) I and II only B) II and III only C) I, II and III only D) I, II and IV only

D) I, II and IV only

Which of the following factors will increase the risk level of an investment? I. a firm's decision to use a high percentage of debt financing II. an economic situation in which consumer prices are rising at a rapid rate III. the ability to trade the investment in a broad market rather than in a thin market IV. unstable currency values A) I and II only B) I, III and IV only C) I and IV only D) I, II and IV only E) II and IV only

D) I, II and IV only

Aggressive mutual funds often employ investing strategies such as I. short selling. II. margin trading. III. option trading. IV. hedging. A) I and III only B) II and III only C) I, II and IV only D) I, II, III and IV

D) I, II, III and IV

An asset allocation plan should consider which of the following factors? I. economic outlook II. capital preservation III. changing investment goals IV. investor risk tolerance A) II only B) II, III and IV only C) I, III and IV only D) I, II, III and IV

D) I, II, III and IV

Investors use mutual funds for which of the following reasons? I. to accumulate wealth II. to minimize risk III. as a speculative vehicle IV. as a storehouse of value A) I and II only B) II, III and IV only C) I, II and IV only D) I, II, III and IV

D) I, II, III and IV

Tax laws allow investors to hold mutual funds in which of the following types of retirement plans? I. ordinary IRAs II. Roth IRAs III. 401-K plans IV. Keogh plans A) I and II only B) II, III and IV only C) I, II and IV only D) I, II, III and IV

D) I, II, III and IV

The efficient market hypothesis rests on which of the following assumptions? I. Information is widely available to all investors almost simultaneously. II. Investors react quickly to new information. III. Investors correctly interpret all available information. IV. Events which affect the market occur randomly. A) I and II only B) I, II and III only C) II, III and IV only D) I, II, III and IV E) I, II and IV only

D) I, II, III and IV

During the 7 year market cycle of 2006-2012, in which category of funds did a majority of funds outperform the market average? A) large cap funds B) small cap funds C) asset allocation funds D) In no category did a majority of funds outperform the market average.

D) In no category did a majority of funds outperform the market average.

The best index to assess the performance of a portfolio diversified among several asset classes such as stocks, bonds and real estate is A) the Lipper Index. B) the NYSE Composite Index. C) the Value Line Index. D) No suitable index exists.

D) No suitable index exists.

Which one of the following statements is correct concerning dollar cost averaging plans? A) Dollar cost averaging is an active trading strategy. B) Dollar cost averaging is a short-term trading strategy. C) The goal of dollar cost averaging is current dividend income. D) The goal of dollar cost averaging is long-term capital appreciation.

D) The goal of dollar cost averaging is long-term capital appreciation.

Assume that $100 is deposited at the end of each year for five years at 10% compound interest and that no withdrawals are made over the five-year period. Based on this data, which one of the following statements is correct? A) The present value will be $310.46. B) The present value can be determined by computing the present value of $500 in five years at 10%. C) The present value will be $500. D) The present value can be determined by computing the present value of a $100 ordinary annuity for five years at 10%. E) The future value will be $550.

D) The present value can be determined by computing the present value of a $100 ordinary annuity for five years at 10%

Which one of the following has the lowest level of risk? A) commercial paper B) banker's acceptance C) money market mutual fund account D) U.S. Treasury bill E) common equity.

D) U.S. Treasury bill

The par or stated value of common stock is important for A) helping the investor determine the stock's intrinsic value. B) helping the board of directors determine the dividend payout. C) helping the market determine the trading price of the stock. D) accounting purposes only. E) determining whether a company is a value or growth stock.

D) accounting purposes only.

Which type of fund is always passively managed? A) a closed-end fund B) a growth fund C) a value fund D) an index fund

D) an index fund

Automatic reinvestment plans A) are a good way to avoid taxes on dividends and capital gains. B) may involve exceptionally high transaction fees. C) do not allow for the purchase of fractional shares. D) are an excellent way to accumulate wealth through disciplined investing.

D) are an excellent way to accumulate wealth through disciplined investing.

Reinvested dividends A) do not increase the value of an investor's holdings. B) are generally sold at a premium over the market price. C) are taxed when the shares purchased with the reinvested dividend are sold. D) are taxed at the time the dividend is paid. E) have the same tax implications as a stock split

D) are taxed at the time the dividend is paid.

The balance sheet value of a firm's assets minus the balance sheet amount of its liabilities is known as A) intrinsic value. B) par value. C) market value. D) book value. E) liquidation value.

D) book value.

Beginning investors with small amounts to invest should A) avoid stock investments completely. B) invest all of their money in one high quality stock. C) buy a portfolio of very low priced stocks (penny stocks). D) buy mutual funds or exchange traded funds (ETFs). E) avoid fixed income investments completely.

D) buy mutual funds or exchange traded funds (ETFs).

The holding period return (HPR) A) reflects only capital gains and losses for investment periods of one year or less. B) calculates the annual dividend yield on stocks or current interest yield on bonds. C) is the most appropriate measure of returns for an investment period exceeding one year. D) can be used to determine the actual total return on stocks, bonds, and other investments for periods of one year or less.

D) can be used to determine the actual total return on stocks, bonds, and other investments for periods of one year or less.

There is evidence to support the contention that company insiders A) cannot earn abnormal profits because they are not permitted to trade shares in their company's stock without a one-month advance notice to the SEC. B) rely primarily on technical indicators to make trading decisions. C) have no distinct advantage when trading shares of their company's stock. D) can profit in a manner that counters the strong form of the efficient market hypothesis. E) generally earn a profit equal to that of public investors.

D) can profit in a manner that counters the strong form of the efficient market hypothesis.

The holding period return for mutual funds should be based on A) net asset value exclusively. B) dividend income exclusively C) capital gains distributions exclusively D) capital gains distributions and dividends.

D) capital gains distributions and dividends.

When interest rates are falling, most of the return on bonds will come from A) interest income. B) risk premiums. C) decreases in default risk. D) capital gains. E) inflation gains.

D) capital gains.

The formula plan that requires maintaining a target dollar investment in the speculative portion of an investor's portfolio is the A) most passive of all the formula plans. B) target return plan. C) constant ratio plan. D) constant dollar plan.

D) constant dollar plan.

To compute the holding period return on a bond investment, the investor should divide the purchase price of the bond into A) any increase or decrease in the bond's price. B) the annual coupon payment. C) the bond's yield to maturity. D) coupon payments received plus or minus any change in the bond's price.

D) coupon payments received plus or minus any change in the bond's price.

The Securities Exchange Act of 1934 A) requires full disclosure of information on all new security issues. B) established trade associations such as the NASD. C) established FINRA as the primary rule making body of the securities industry. D) created the SEC as the regulator of the securities exchanges. E) authorized the SEC to regulate mutual funds.

D) created the SEC as the regulator of the securities exchanges.

In a severe recession, the major source of risk faced by investors who purchase corporate bonds is A) tax risk. B) liquidity risk. C) purchasing power risk. D) default risk. E) interest rate risk.

D) default risk.

Mutual fund investors delegate all of the following decisions to the fund's managers EXCEPT A) which companies and industries to invest in. B) when to buy and sell individual stocks. C) how many securities to hold in the portfolio. D) how to allocate investments among different classes of assets such as stocks, bonds, cash and real estate.

D) how to allocate investments among different classes of assets such as stocks, bonds, cash and real estate.

The stock of a technology company has an expected return of 15% and a standard deviation of 20% The stock of a pharmaceutical company has an expected return of 13% and a standard deviation of 18%. A portfolio consisting of 50% invested in each stock will have an expected return of 14 % and a standard deviation A) the answer cannot be determined with the information given. B) greater than the average of 20% and 18%. C) the average of 20% and 18%. D) less than the average of 20% and 18%.

D) less than the average of 20% and 18%.

An investor who wants to use mutual funds as a storehouse of value should invest in A) high-yield corporate bond funds and growth funds. B) growth funds and equity-income funds. C) equity-income funds and long-term bond funds. D) money funds and short-term bond funds.

D) money funds and short-term bond funds.

For which one of the following situations will the price-to-sales valuation model work but the dividend and cash flow models will not? A) top-performing firm in a mature industry B) water-powered electric utility company C) mature firm with minimal growth opportunities D) newly-formed biotechnology company with negative earnings

D) newly-formed biotechnology company with negative earnings

Under the variable-ratio plan, additional speculative investments are made when the ratio A) of conservative investments to speculative investments increases by 10%. B) of the rate of return on the speculative investments exceeds the overall market return by 1% or more. C) of the realized rate of return falls below the desired rate of return by 1% or more. D) of the value of the speculative investments to the total portfolio value drops below a predetermined level.

D) of the value of the speculative investments to the total portfolio value drops below a predetermined level.

The risk-return tradeoff of four diversified portfolios is described in the following figure: If you were a hedge fund manager interested in earning a positive return without taking on systematic risk, you should A) purchase portfolio C and short an equal proportion of portfolio D. B) short portfolio D. C) purchase portfolio A and short an equal proportion of portfolio C. D) purchase portfolio B and short an equal proportion of portfolio D. E) purchase portfolio A.

D) purchase portfolio B and short an equal proportion of portfolio D.

Investors who seek triple (federal, state, and local) tax-free income should invest in ________ bond funds. A) convertible B) indexed C) mortgage-backed D) single-state municipal

D) single-state municipal

Which of the following practices is prohibited by the Insider Trading and Fraud Act of 1988? A) selling of stock by officers of the company B) private sales of stock between executives of the company C) promoting a low-price stock, sometimes with false information, in the hopes of selling after a price spike D) the use of nonpublic information to make profitable stock transactions E) the granting of stock options to corporate executives in lieu of salaries

D) the use of nonpublic information to make profitable stock transactions

The formula plan which requires the greatest management attention and is also the most aggressive is called the ________ plan. A) dollar cost averaging B) constant dollar C) constant ratio D) variable ratio

D) variable ratio

An investor adopts a policy of investing in both an aggressive mutual fund and a short-term bond fund. When the value of the aggressive fund exceeds 65% of the portfolio value, shares of that fund are sold such that the aggressive fund represents only 45% of the portfolio. This is an example of a ________ plan. A) constant-dollar B) dollar-cost averaging C) constant-ratio D) variable-ratio

D) variable-ratio

An aggressive growth mutual fund is least likely to purchase a stock A) with a high P/E ratio. B) with a high anticipated rate of growth. C) of an unseasoned firm. D) with a high dividend yield.

D) with a high dividend yield.

The present value of $1,000, discounted at the rate of 5% per year, to be received at the end of 3 years is equal to A) $1,000 × (1.05) × (1.05) × (1.05) B) $1,000 - ($1,000) × .03 × 5. C) $1,000 × (1.05)3. D) $1,000/(1.03)5. E) $1,000/(1.05)3.

E) $1,000/(1.05)3.

Andrew and Jennifer are in the 25% marginal tax bracket. Three years ago they purchased 100 shares of stock at $20 a share. In 2012, they sold the 100 shares for $29 a share. What is the amount of federal income tax they owe as a result of this sale? A) $165 B) $435 C) $225 D) $180 E) $135

E) $135

Taylor has saved $400 at the end of every month for the last 4 years with the intention of paying cash for a new car. She has earned a fixed annual rate of 4% over the 4 year period; interest is compounded monthly. How much can she pay for her new car at the end of the fourth year? A) $1,699 B) $22,272 C) $17,716 D) $55,705 E) $20,784

E) $20,784

DMC3 Inc. will pay no dividend for 2016 or 2017. At the end of 2018, it will pay a dividend of $1.50. Thereafter dividends will grow at 4% per year. The required rate of return is 10%. The intrinsic value of DMC3 shares is (assume you are at the beginning of 2016) A) $22.72. B) $24.91. C) $34.61. D) $26.00 E) $20.66.

E) $20.66.

Martin's Inc. is expected to pay annual dividends of $2.50 a share for the next three years. After that, dividends are expected to increase by 3% annually. What is the current value of this stock to you if you require a 9% rate of return on this investment? A) $42.92 B) $41.81 C) $40.11 D) $36.73 E) $39.47

E) $39.47

Lindor Inc.'s $100 par value preferred stock pays a dividend fixed at 8% of par. To earn 12% on an investment in this stock, you need to purchase the shares at a per share price of A) $150.00. B) $96.00. C) $9.60. D) $8.00. E) $66.67

E) $66.67

Melissa owns the following portfolio of stocks. What is the return on her portfolio? A) 9.0% B) 11.5% C) 8.0% D) 10.9% E) 9.8%

E) 9.8%

Which one of the following statements concerning interest rates is correct? A) Economic expansions will cause interest rates to decline. B) A slow increase in the money supply will cause interest rates to rise. C) A swift increase in the size of the Federal Reserve balance sheet will lower inflation. D) Rising interest rates in foreign countries will cause U.S. interest rates to decline. E) A federal budget surplus will cause interest rates to decline.

E) A federal budget surplus will cause interest rates to decline.

Which one of the following statements about the NYSE is correct? A) Each member of the exchange owns a trading post. B) Any listed stock may be traded at any of 20 trading posts. C) Brokerage firms are only permitted to have one individual trading on the floor of the exchange. D) Most trades for NYSE listed stocks occur on the trading floor. E) Buy orders are filled at the lowest price and sell orders are filled at the highest price.

E) Buy orders are filled at the lowest price and sell orders are filled at the highest price.

Which of the following are true about stock market returns as measured by the S&P 500 index? I. In 2008 alone stocks in the index lost between 30% and 40% of their value. II. $10,000 invested in the index in March 2009 would have been worth more than $20,000 by the end of 2016. III. From the beginning of 2000 to the end of 2010, the index more than doubled in value. IV. Both stock and real estate prices recovered strongly in the period between early 2009 and late 2014. A) I, and IV only B) II, III and IV only C) II and IV only D) I II and III only E) I ,II and IV only

E) I ,II and IV only

The Capital Asset Pricing Model (CAPM) includes which of the following in its base assumptions? I. Investors should earn a minimum return equal to the risk-free rate. II. Investors in the market should earn a return greater than the return on the overall market. III. Investors should be rewarded for the amount of risk they assume. IV. Investors cannot earn a return located below the Security Market Line. A) I, III and IV only B) I and II only C) II and IV only D) I, II and III only E) I and III only

E) I and III only

To determine whether a pharmaceutical company's profitability ratios indicate strength or weakness, we should I. compare them to others in the same industry. II. compare them to companies in unrelated industries such as energy or banking. III. compare them to previous years. IV. compare them to absolute standards established by the CFA Institute. A) IV only B) I and II only C) III and IV only D) III only E) I and III only

E) I and III only

Which of the following guidelines are appropriate for inclusion in a portfolio management policy? I. Diversify among different types of securities and across industry and geographic lines. II. Determine the risk level and financial situation of the individual investor. III. Utilize beta to help align the portfolio to the risk level of the investor. IV. Minimize the standard deviation of each security in the portfolio. A) I, II, III and IV B) I and III only C) II, III and IV only D) I, II and IV only E) I, II and III only

E) I, II and III only

Which of the following are characteristics of short selling? I. borrowing shares of stock from a brokerage firm or other investors II. selling shares of stock you do not own III. betting the stock price will increase IV. limiting losses per share to the price at which the stock was sold A) III and IV only B) I, II, III and IV C) I, II and IV only D) I and II only E) I, II, III only

E) I, II, III only

Which of the following characteristics are referred to as representativeness? I. hesitating to sell stocks at a loss II. basing conclusions on small samples III. underestimating the effects of random chance IV. underestimating the level of risk in an investment A) I and IV only B) I, II and III only C) I, II, III and IV D) II, III and IV only E) II and III only

E) II and III only

Which one of the following statements is correct? A) The market responds immediately to reflect the information contained in quarterly earnings reports. B) Large, well-known stocks tend to outperform smaller, more obscure stocks on a risk-adjusted basis. C) The weekend effect states that security prices tend to rise between Friday afternoon and Monday morning. D) The market fully anticipates the information contained in an earnings announcement prior to the actual announcement. E) Low P/E stocks tend to outperform high P/E stocks on a risk-adjusted basis.

E) Low P/E stocks tend to outperform high P/E stocks on a risk-adjusted basis.

Typical hedge funds are characterized by A) their appeal to a large number of small investors. B) investment portfolios consisting only of traditional stocks and bonds. C) a fee structure that only pays managers when the fund makes money. D) low minimum investment requirements. E) active investment strategies.

E) active investment strategies.

An investment paying 4% compounded quarterly will have a value at the end of one year equal to A) an investment paying 1% compounded quarterly at the end of 4 years. B) an investment paying 4% compounded annually at the end of 4 years. C) an investment paying 2% compounded semi-annually at the end of 1 year. D) an investment paying 16% compounded annually at the end of 1 year. E) an investment paying 1% compounded annually at the end of 4 years.

E) an investment paying 1% compounded annually at the end of 4 years.

The published analysis and recommendations of an individual brokerage firm is called a A) investment newsletter. B) comparative data source. C) broker's subscription report. D) prospectus. E) back-office research report.

E) back-office research report.

Crossing markets are those that A) conduct business at locations in varying time zones. B) trade foreign securities. C) conduct transactions between institutional and individual traders. D) rely on a visible limit order book. E) fill only the orders which have opposing orders at identical prices.

E) fill only the orders which have opposing orders at identical prices.

An individual investor who wishes to borrow money to buy stocks must open a A) custodial account. B) signature account. C) joint account. D) PayPal account. E) margin account.

E) margin account.

A rights offering is the A) sale of securities directly to a select group of investors. B) sale of newly issued shares of stock to the general public with voting rights included. C) sale of newly issued shares of stock to the general public with voting rights withheld. D) secondary offering of securities to the general public. E) offering of new securities to current shareholders on a pro-rata basis.

E) offering of new securities to current shareholders on a pro-rata basis.

Lee is considering buying one of two newly-issued bonds. Bond A is a twenty-year, 7.5% coupon bond that is non-callable. Bond B is a twenty-year, 8.25% bond that is callable after two years. Both bonds are comparable in all other aspects. Lee plans on holding his bond to maturity. What should Lee do if he feels that interest rates are going to decline by 2% in the near future and then remain relatively stable thereafter? A) purchase Bond B B) negotiate a higher rate on Bond A C) purchase neither A nor B at this time D) we cannot determine from the information given E) purchase Bond A

E) purchase Bond A

Investors who buy mutual funds that have had large gains over the last few years are exhibiting a tendency known as A) winning. B) overconfidence. C) loss aversion. D) narrow framing. E) representativeness.

E) representativeness.

American depositary shares (ADS) are A) Tier 1 capital invested by US commercial banks. B) foreign currency deposits in American banks. C) American currency deposits in foreign banks D) shares of American companies traded on foreign markets. E) shares of foreign companies traded on the U.S. markets.

E) shares of foreign companies traded on the U.S. markets.

One of the calendar effect market anomalies indicates that ________ in value during January. A) large cap stocks tend to decline B) strong performers from the previous year tend to decline C) equities in general tend to increase D) equities in general tend to decline E) small cap stocks tend to increase

E) small cap stocks tend to increase

Which one of the following is an example of an annuity? A) the recept of a dividend that grows by 2% each quarter forever B) the receipt of $100 a month for three months and then $150 a month for two months C) the receipt of $50 in January, March, April, June, August, September and December D) the payment of $389 in January, $200 in February, and $200 in March E) the payment of $259 a month for three consecutive years

E) the payment of $259 a month for three consecutive years


Conjuntos de estudio relacionados

CH. 13: Return, Risk, and the Security Market Line

View Set

PAC and interest group AP classroom

View Set

Chapter 30 Alterations in Immune Function

View Set

Peer Tutoring Analogies Quiz set

View Set

Section 16: Real Estate Financing: Mortgage Loans

View Set